LSAT and Law School Admissions Forum

Get expert LSAT preparation and law school admissions advice from PowerScore Test Preparation.

 Administrator
PowerScore Staff
  • PowerScore Staff
  • Posts: 8950
  • Joined: Feb 02, 2011
|
#27928
Complete Question Explanation

Weaken. The correct answer choice is (A)

According to this stimulus, corporations that encourage frequent social events in the workplace show higher profits than those businesses that rarely encourage such events. The writer concludes, based upon this, that EZ Corp. could raise its profits by having more staff parties during business hours. This conclusion is apparently based on the author’s questionable assumption that there must be a cause and effect relationship: increased socializing in the workplace causes higher profits.

The question stem asks which answer choice most weakens the conclusion.
To weaken a causal conclusion, we have several options to consider:
  • 1. ..... Show that an alternative cause exists.
    2. ..... Show that the supposed cause exists without the supposed effect.
    3. ..... Show that the hypothesized effect exists without the assumed cause.
    4. ..... Show that the elements thought to be cause and effect were reversed.
    5. ..... Show an effective attack on the data.
Answer choice (A): This is the correct answer choice. If the great majority of corporations that encourage frequent social events in the workplace do so because they are already earning higher profits, this appears to be a case of mistaken cause and effect (as prephrased fourth above).

Answer choice (B): The writer’s conclusion is that staff parties during business hours will lead to higher profits. The fact that there are some corporations with higher profits who throw parties after hours is not relevant to the value of business hour staff parties, so this choice is incorrect.

Answer choice (C): Some might be tempted to equate the “above average profits” in this answer choice with boosting profits as concluded in the stimulus. These are different effects, and therefore, this answer choice does not weaken the conclusion in the stimulus which deals with boosting profits, not profits that happen to be “above average.”

Answer choice (D): This answer choice states that frequent social events in a corporate workplace leave employees with less time to perform their assigned duties than they otherwise would have. This does not address why companies with frequent workplace social activities tend to have higher than average profits. Because it does not address this, it cannot be said to weaken the writer’s conclusion.

Answer choice (E): This answer choice does not present an alternative cause, but it may show that the cause (workplace parties) has existed in the past. The stimulus states that EZ Corporation has not always been one of the most profitable companies for its size. But how did it compare to other similar companies at that time? Were these parties boosting the corporation’s profitability? Since we don’t know, this limited information has little effect on the strength of the argument in the stimulus.
 brittany1990
  • Posts: 21
  • Joined: Jul 11, 2012
|
#5745
After reading the stimulus and question stem, I realized that there was a causal relationship (social events cause higher profits). I eliminated B,D,and E so I was left with A and C. I picked C because I thought it was showing an effect without a cause (higher profits without social events).

I read the answer descriptions online and it said C is incorrect because "above average profits" does not equate to "higher profits." I understand that, but then when I re-looked at answer choice A it also says "above-average profits" so I'm still confused.

Thanks!
Brittany
 Steve Stein
PowerScore Staff
  • PowerScore Staff
  • Posts: 1153
  • Joined: Apr 11, 2011
|
#5750
Hey Brittany,

Thanks for your question. In that one, the author points to a correlation between social events and higher profits, and recommends that EZ Corp have more social events.

As you correctly point out, the author appears to believe that the social events are causing the higher profits. The stimulus is followed by a weaken question, so the correct answer choice will provide reason to doubt the causal hypothesis advanced by the author.

Answer choice A provides that most of the companies having the social events were doing so in part because they were already enjoying high profits. If higher profits preceded the social events in most of these companies, that kills the author's argument.

Answer choice C provides that the EZ already has higher than average profits. This doesn't hurt the author, whose argument is that EZ can boost its profits by having more social events. Since a company can boost its profits even if it is already very profitable, this choice doesn't really hurt the author's argument.

I hope that's helpful! Let me know whether that clears this one up--thanks!

~Steve
 brittany1990
  • Posts: 21
  • Joined: Jul 11, 2012
|
#5783
Got it, thank you!
 cecilia
  • Posts: 66
  • Joined: Nov 07, 2011
|
#18149
HI all - To me D was an extremely attractive answer. Could anyone explain in general terms, how D is incorrect and how it does not weaken the argument? Thank you in advance......
 Steve Stein
PowerScore Staff
  • PowerScore Staff
  • Posts: 1153
  • Joined: Apr 11, 2011
|
#18157
Hey Cecilia,

Good question! In that one, to review, the stimulus provides that companies with more workplace social events make greater profits than thsoe who rarely have such events. Therefore, concludes the author, EZ Co could boost profits by having more parties during business hours.

The question that follows asks for the answer choice that most weakens the author's argument. Answer choice (D) provides that frequent social events leave employees with less time to do their assigned tasks. This is an appealing answer choice, because it seems to provide a reason to object to such events...but it doesn't weaken the argument that EZ Co. could boost profits by doing so, based on the experience of other companies that do so.

Very appealing answer choice! Let me know whether this is clear--thanks!

Steve
 cecilia
  • Posts: 66
  • Joined: Nov 07, 2011
|
#18158
Hi Steve and thanks so much for the prompt response. It does clear things up.

I was trying to think of why D was so tempting, as I seem to repeat mistakes like that on weaken questions - - as in picking answer choices because they seem to clearly attack the premise. I believe you said it was wrong because it "does not weaken the argument that EZ should...". Can one say, then, that it is also wrong because it attacks the premise and not the argument itself?

Thanks.
 Nikki Siclunov
PowerScore Staff
  • PowerScore Staff
  • Posts: 1362
  • Joined: Aug 02, 2011
|
#18163
Hi cecilia,

Let me jump in real quick. Indeed, answer choice (D) appears to undermine the premise of the argument, namely, that there is a correlation between parties and profits. Note, however, that a correlation may still exist even if such parties hurt employee productivity. Ultimately, answer choice (D) is incorrect because it does not address the unwarranted assumption that staff parties are actually responsible for - i.e. causally contribute to - higher profits.

Does that make sense?

Thanks!
 cecilia
  • Posts: 66
  • Joined: Nov 07, 2011
|
#18165
Brilliant! Thanks Nikki !
 livgrala13
  • Posts: 2
  • Joined: Aug 04, 2020
|
#77732
Hi!
So, I see your explanation why C) does not weaken the argument by providing the same effect without the cause since above average profits do not equal a boost in profits, and therefore, do not yield the same result. However, the correct answer choice A also uses the term above average profits in reversing the relationship. Although I see how A reverses the relationship, I am confused why it's not wrong for the same reason.

Get the most out of your LSAT Prep Plus subscription.

Analyze and track your performance with our Testing and Analytics Package.